Difference between revisions of "2020 AIME I Problems/Problem 1"
(→Solution 2) |
(→Solution 1) |
||
Line 25: | Line 25: | ||
<math>\frac{AQ}{CD} = \frac{735}{847 + RC} = \frac{RC - 112}{RC}</math><br /> | <math>\frac{AQ}{CD} = \frac{735}{847 + RC} = \frac{RC - 112}{RC}</math><br /> | ||
− | <math>735RC = (RC + | + | <math>735RC = (RC + 17797)(RC - 13)</math><br /> |
− | <math>0 = RC^2 - | + | <math>0 = RC^2 - 13\cdot17797</math> |
</div> | </div> | ||
− | Thus, <math> RC = \sqrt{ | + | Thus, <math> RC = \sqrt{13*17797} = 308</math>. |
=== Solution 2 === | === Solution 2 === | ||
We have <math>\triangle BRQ\sim \triangle DRC</math> so <math>\frac{112}{RC} = \frac{BR}{DR}</math>. We also have <math>\triangle BRC \sim \triangle DRP</math> so <math>\frac{ RC}{847} = \frac {BR}{DR}</math>. Equating the two results gives <math>\frac{13}{RC} = \frac{ RC}{17797}</math> and so <math>RC^2=13*17797</math> which solves to <math>RC=\boxed{481}</math> | We have <math>\triangle BRQ\sim \triangle DRC</math> so <math>\frac{112}{RC} = \frac{BR}{DR}</math>. We also have <math>\triangle BRC \sim \triangle DRP</math> so <math>\frac{ RC}{847} = \frac {BR}{DR}</math>. Equating the two results gives <math>\frac{13}{RC} = \frac{ RC}{17797}</math> and so <math>RC^2=13*17797</math> which solves to <math>RC=\boxed{481}</math> |
Revision as of 15:59, 27 February 2020
Contents
Problem
Let be a parallelogram. Extend through to a point and let meet at and at Given that and find
Solution
Solution 1
There are several similar triangles. , so we can write the proportion:
Also, , so:
Substituting,
Thus, .
Solution 2
We have so . We also have so . Equating the two results gives and so which solves to